The proof is very nice, but one needs to be absolutely clear about what needs to be proven in order to understand it. The claim is, if $\lambda_i : \varprojlim D \to D_i$ is a limiting cone in $\mathcal{D}$, then $G \lambda_i : G(\varprojlim D) \to G D_i$ is a limiting cone in $\mathcal{C}$. We do not postulate the existence of $\varprojlim G D$; this is what we are going to prove.

So suppose we are given a cone $\mu_i : X \to G D_i$ in $\mathcal{C}$. This yields a cone of hom-sets $$(\mu_i)_* : \mathcal{C}(A, X) \to \mathcal{C}(A, G D_i)$$ and since $\varprojlim \mathcal{C}(A, G D_i) \cong \mathcal{C}(A, G(\varprojlim D))$ naturally in $A$ (by the argument you cited), by the Yoneda lemma it follows that there is a unique natural transformation $\varphi_* : \mathcal{C}(-, X) \Rightarrow \mathcal{C}(-, G(\varprojlim D))$ such that $$(\mu_i)_* = (G \lambda_i)_* \circ \varphi_*$$ where $\varphi_*$ comes from a morphism $\varphi : X \to G(\varprojlim D)$. Thus $G \lambda_i : G(\varprojlim D) \to G D_i$ is indeed a limiting cone.


This is essentially taken from the proof of Proposition 2.4.5 p. 36 in these notes by Pierre Schapira.

Let me use the abbreviation $$ L:=\lim_{\underset{i}{\longleftarrow}}\quad. $$

Let $C$ and $D$ be categories, let $b:I^{op}\to C$, $i\mapsto b_i$, be a projective system, let $Lb$ be its limit (we assume it exists), let $F:C\to D$ be a functor, let $G:D\to C$ be its left adjoint (we assume it exists), and let $y$ be an object of $D$.

We have the following commuting squares of morphisms and isomorphisms, where the vertical arrows are induced by the $i$ th canonical projections: $$ \begin{matrix} D(y,FLb)&\simeq&C(Gy,Lb)\\ \downarrow&&\downarrow\\ D(y,Fb_i)&\simeq&C(Gy,b_i), \end{matrix} $$

$$ \begin{matrix} C(Gy,Lb)&\simeq&LC(Gy,b)\\ \downarrow&&\downarrow\\ C(Gy,b_i)&=&C(Gy,b_i), \end{matrix} $$

$$ \begin{matrix} LC(Gy,b)&\simeq&LD(y,Fb)\\ \downarrow&&\downarrow\\ C(Gy,b_i)&\simeq&D(y,Fb_i), \end{matrix} $$

$$ \begin{matrix} LD(y,Fb)&\simeq&D(y,LFb)\\ \downarrow&&\downarrow\\ D(y,Fb_i)&=&D(y,Fb_i). \end{matrix} $$ By splicing these squares, we get the following commuting square of morphisms and isomorphisms: $$ \begin{matrix} D(y,FLb)&\simeq&D(y,LFb)\\ \downarrow&&\downarrow\\ D(y,Fb_i)&=&D(y,Fb_i), \end{matrix} $$ which is what we wanted.

EDIT A. Let's go back to the first square: $$ \begin{matrix} D(y,FLb)&\simeq&C(Gy,Lb)\\ \downarrow&&\downarrow\\ D(y,Fb_i)&\simeq&C(Gy,b_i). \end{matrix} $$ The isomorphisms are given by the adjunction. If $p_i:Lb\to b_i$ denotes the $i$ th canonical projection, then the first downward arrow is $D(y,Fp_i)$, and the second is $C(Gy,p_i)$. To show that the square commutes, we only need to invoke the fact that the adjunction is functorial in the second variable.

Now to the second square: $$ \begin{matrix} C(Gy,Lb)&\simeq&LC(Gy,b)\\ \downarrow&&\downarrow\\ C(Gy,b_i)&=&C(Gy,b_i). \end{matrix} $$ By assumption, we have chosen a representing object $Lb$ and an isomorphism $$ C(x,Lb)\simeq LC(x,b) $$ functorial in $x\in\text{Ob}(C)$. We have a natural map from $LC(x,b)$ to $C(x,b_i)$ --- because $LC(x,b)$ is a projective limit of sets. Then we define the map from $C(x,Lb)$ to $C(x,b_i)$ as the one which makes the above square commutative. All this being functorial in $x$, the Yoneda Lemma yields the morphism $p_i:Lb\to b_i$ used above.

EDIT B. The third and fourth squares are handled similarly. So we end up with the square $$ \begin{matrix} D(y,FLb)&\simeq&D(y,LFb)\\ \downarrow&&\downarrow\\ D(y,Fb_i)&=&D(y,Fb_i), \end{matrix} $$ which commutes for all $i$. What we want to prove is the existence of an isomorphism $FLb\simeq LFb$ such that the square $$ \begin{matrix} FLb&\simeq&LFb\\ \downarrow&&\downarrow\\ Fb_i&=&Fb_i \end{matrix} $$ commutes for all $i$. But, in view of Yoneda, the above square commutes because the previous one does.

EDIT C. Alternative wording of the poof that $$ \begin{matrix} C(x,Lb)&\simeq&LC(x,b)\\ \downarrow&&\downarrow\\ C(x,b_i)&=&C(x,b_i) \end{matrix} $$ commutes:

A morphism $f\in C(x,Lb)$ is given by a family $f_\bullet=(f_j)_{j\in I}\in LC(x,b)$ satisfying the obvious compatibility conditions, and we have $f_j=p_j\circ f$ for all $j$. So, $f$ and $f_\bullet$ correspond under the isomorphism in the above square. Moreover, the first vertical arrow maps $f$ to $f_i$, and the second vertical arrow maps $f_\bullet$ to $f_i$.


Awodey sometimes hand waves his proofs, especially the ones in which you have to get your hands dirty--for instance, his proof that every presheaf is a colimit of representables.

Anyway, in this case, application of Yoneda is overkill because it is no harder to prove RAPL just by using adjointness:

Suppose $\left \langle L,\lambda _{i} \right \rangle$ is a limit cone to $D$. Then $\left \langle GL,G\lambda _{i} \right \rangle$ is a cone to $GD$. Suppose $\left \langle X,\mu _{i} \right \rangle$ is a cone to $GD$. Then taking adjoints, we get a cone to $D$, namely, $\left \langle FX,\mu _{i}^{*}\right \rangle$. Since $\left \langle L,\lambda _{i} \right \rangle$ is a limit cone, we get an arrow $\phi ^{*}:FX\rightarrow L$ unique with property that $\lambda _{i}\circ \phi ^{*}=\mu ^{*}_{i}$. Taking adjoints again, it is easy to see that $\phi :X\rightarrow GL$ is the unique arrow making the required triangle commute, for

if we write \begin{array}{ccc} \hom( X,GL) & \rightarrow & \hom (FX,L) \\ \downarrow & & \downarrow \\ \hom(X,GD _{i}) & \rightarrow & \hom(FX,D_{1} ) \end{array} and follow $\phi $ around the square, we see that $\lambda _{i}\circ \phi ^{*} =(G\lambda _{i}\circ \phi )^{*}$. But LHS of this is just $\mu ^{*}_{i}$, so that $G\lambda _{i}\circ \phi =\mu_{i} $ and $\phi $ is unique because $\phi ^{*}$ is.


A new answer was added recently, and from my perspective, none of the answers reflect what I think the proof is trying to get at, so I thought I'd add another answer. However, I do agree with Zhen Lin's answer (+1) that the proof goes wrong when it assumes already that the limit of $GD$ exists. The point is rather to show that $(G\lim D,G(\alpha_i))$ form a limiting cone if $(\lim D,\alpha_i)$ form a limiting cone for $D$.

Other answers argue that there are better proofs, and I won't necessarily agree or disagree, but the idea of this proof is sound, so let me see if I can salvage it.

Review of the Yoneda Lemma

The key point is that part of the Yoneda lemma says that natural transformations from a Yoneda functor $\newcommand\C{\mathcal{C}}\newcommand\D{\mathcal{D}}\C(-,X)$ to another functor $F:\C^{\text{op}}\to \newcommand\Set{\mathbf{Set}}\Set$ are naturally isomorphic to the elements of $F(X)$.

In particular, if you have a natural isomorphism $$\C(-,L)\simeq \text{cones from $-$ to }D,$$ the natural isomorphism corresponds to a cone from $L$ to $D$, given by the image of $1_L\in\C(L,L)$, which is the limiting cone.

The argument

Thus, the way this argument should go is the following: Let $L=\lim D$ for notational convenience. $$ \begin{align} \C(-,GL)&\simeq \D(F-,L) \\ &\simeq \lim_i \D(F-,D_i)\\ &\simeq \lim_i \D(-,GD_i)\\ &\simeq \text{cones from $-$ to } GD. \end{align} $$ This natural isomorphism proves that $GL$ is the limit of $GD$ already. To see what the limiting cone is, we have to see where $1_{GL}$ maps under the natural isomorphism.

Under the first natural isomorphism, $1_{GL}$ maps to the [counit] of the adjunction, $\epsilon_L : FGL\to L$. The next natural isomorphism sends $\epsilon_L$ to $(\alpha_i\circ \epsilon_L)_i$, which belong to the limit of hom sets because the $(\alpha_i)$ do. Finally, the adjunct of $\alpha_i\circ \epsilon_L$ is given by $$G(\alpha_i\circ \epsilon_L)\circ \eta_{GL} = G\alpha_i \circ G\epsilon_L \circ \eta_{GL} =G\alpha_i,$$ with the last equality being the triangle identities.

Finally, the last isomorphism is just a reinterpretation of the elements of the limit as cones, since checking what it means to take the limit in $\Set$ of the hom-sets against what it means for elements to be cones we see that they agree. Thus $(G\alpha_i)$ is the limiting cone corresponding to this natural isomorphism, as desired.

TL;DR

The natural isomorphism already encodes the data of the limiting cone. All we need to do is check that we get the expected limiting cone.